Is there a way to find this limit algebraically? $limlimits_{xto infty} frac{x}{sqrt{x^2 + 1}}$












6












$begingroup$


I'm a Calculus I student and my teacher has given me a set of problems to solve with L'Hoptial's rule. Most of them have been pretty easy, but this one has me stumped.



$$limlimits_{xto infty} frac{x}{sqrt{x^2 + 1}}$$



You'll notice that using L'Hopital's rule flips the value of the top to the bottom. For example, using it once returns:



$$limlimits_{xto infty} frac{sqrt{x^2 + 1}}{x}$$



And doing it again returns you to the beginning:



$$limlimits_{xto infty} frac{x}{sqrt{x^2 + 1}}$$



I of course plugged it into my calculator to find the limit to evaluate to 1, but I was wondering if there was a better way to do this algebraically?










share|cite|improve this question











$endgroup$








  • 6




    $begingroup$
    $frac{x}{sqrt{x^2 + 1}}=frac{sqrt{x^2}}{sqrt{x^2+1}}=sqrt{frac{x^2}{x^2+1}}$
    $endgroup$
    – Mason
    Jan 5 at 18:41






  • 3




    $begingroup$
    The "right" way to solve this is as Thomas Shelby wrote in his answer. However, you can almost intuitively see that the limit is $1$ even without using a calculator. Indeed, if $x$ is a huge positive number, say $10^{5863}$, the difference between $x^2$ and $x^2 + 1$ is extremely tiny. So the expression is essentially indistinguishable from ${xoversqrt{x^2}} = {xover x} = 1$.
    $endgroup$
    – Andreas Rejbrand
    Jan 5 at 19:37


















6












$begingroup$


I'm a Calculus I student and my teacher has given me a set of problems to solve with L'Hoptial's rule. Most of them have been pretty easy, but this one has me stumped.



$$limlimits_{xto infty} frac{x}{sqrt{x^2 + 1}}$$



You'll notice that using L'Hopital's rule flips the value of the top to the bottom. For example, using it once returns:



$$limlimits_{xto infty} frac{sqrt{x^2 + 1}}{x}$$



And doing it again returns you to the beginning:



$$limlimits_{xto infty} frac{x}{sqrt{x^2 + 1}}$$



I of course plugged it into my calculator to find the limit to evaluate to 1, but I was wondering if there was a better way to do this algebraically?










share|cite|improve this question











$endgroup$








  • 6




    $begingroup$
    $frac{x}{sqrt{x^2 + 1}}=frac{sqrt{x^2}}{sqrt{x^2+1}}=sqrt{frac{x^2}{x^2+1}}$
    $endgroup$
    – Mason
    Jan 5 at 18:41






  • 3




    $begingroup$
    The "right" way to solve this is as Thomas Shelby wrote in his answer. However, you can almost intuitively see that the limit is $1$ even without using a calculator. Indeed, if $x$ is a huge positive number, say $10^{5863}$, the difference between $x^2$ and $x^2 + 1$ is extremely tiny. So the expression is essentially indistinguishable from ${xoversqrt{x^2}} = {xover x} = 1$.
    $endgroup$
    – Andreas Rejbrand
    Jan 5 at 19:37
















6












6








6





$begingroup$


I'm a Calculus I student and my teacher has given me a set of problems to solve with L'Hoptial's rule. Most of them have been pretty easy, but this one has me stumped.



$$limlimits_{xto infty} frac{x}{sqrt{x^2 + 1}}$$



You'll notice that using L'Hopital's rule flips the value of the top to the bottom. For example, using it once returns:



$$limlimits_{xto infty} frac{sqrt{x^2 + 1}}{x}$$



And doing it again returns you to the beginning:



$$limlimits_{xto infty} frac{x}{sqrt{x^2 + 1}}$$



I of course plugged it into my calculator to find the limit to evaluate to 1, but I was wondering if there was a better way to do this algebraically?










share|cite|improve this question











$endgroup$




I'm a Calculus I student and my teacher has given me a set of problems to solve with L'Hoptial's rule. Most of them have been pretty easy, but this one has me stumped.



$$limlimits_{xto infty} frac{x}{sqrt{x^2 + 1}}$$



You'll notice that using L'Hopital's rule flips the value of the top to the bottom. For example, using it once returns:



$$limlimits_{xto infty} frac{sqrt{x^2 + 1}}{x}$$



And doing it again returns you to the beginning:



$$limlimits_{xto infty} frac{x}{sqrt{x^2 + 1}}$$



I of course plugged it into my calculator to find the limit to evaluate to 1, but I was wondering if there was a better way to do this algebraically?







calculus






share|cite|improve this question















share|cite|improve this question













share|cite|improve this question




share|cite|improve this question








edited Jan 5 at 21:10









Blue

47.9k870152




47.9k870152










asked Jan 5 at 18:36









Jae SwanepoelJae Swanepoel

311




311








  • 6




    $begingroup$
    $frac{x}{sqrt{x^2 + 1}}=frac{sqrt{x^2}}{sqrt{x^2+1}}=sqrt{frac{x^2}{x^2+1}}$
    $endgroup$
    – Mason
    Jan 5 at 18:41






  • 3




    $begingroup$
    The "right" way to solve this is as Thomas Shelby wrote in his answer. However, you can almost intuitively see that the limit is $1$ even without using a calculator. Indeed, if $x$ is a huge positive number, say $10^{5863}$, the difference between $x^2$ and $x^2 + 1$ is extremely tiny. So the expression is essentially indistinguishable from ${xoversqrt{x^2}} = {xover x} = 1$.
    $endgroup$
    – Andreas Rejbrand
    Jan 5 at 19:37
















  • 6




    $begingroup$
    $frac{x}{sqrt{x^2 + 1}}=frac{sqrt{x^2}}{sqrt{x^2+1}}=sqrt{frac{x^2}{x^2+1}}$
    $endgroup$
    – Mason
    Jan 5 at 18:41






  • 3




    $begingroup$
    The "right" way to solve this is as Thomas Shelby wrote in his answer. However, you can almost intuitively see that the limit is $1$ even without using a calculator. Indeed, if $x$ is a huge positive number, say $10^{5863}$, the difference between $x^2$ and $x^2 + 1$ is extremely tiny. So the expression is essentially indistinguishable from ${xoversqrt{x^2}} = {xover x} = 1$.
    $endgroup$
    – Andreas Rejbrand
    Jan 5 at 19:37










6




6




$begingroup$
$frac{x}{sqrt{x^2 + 1}}=frac{sqrt{x^2}}{sqrt{x^2+1}}=sqrt{frac{x^2}{x^2+1}}$
$endgroup$
– Mason
Jan 5 at 18:41




$begingroup$
$frac{x}{sqrt{x^2 + 1}}=frac{sqrt{x^2}}{sqrt{x^2+1}}=sqrt{frac{x^2}{x^2+1}}$
$endgroup$
– Mason
Jan 5 at 18:41




3




3




$begingroup$
The "right" way to solve this is as Thomas Shelby wrote in his answer. However, you can almost intuitively see that the limit is $1$ even without using a calculator. Indeed, if $x$ is a huge positive number, say $10^{5863}$, the difference between $x^2$ and $x^2 + 1$ is extremely tiny. So the expression is essentially indistinguishable from ${xoversqrt{x^2}} = {xover x} = 1$.
$endgroup$
– Andreas Rejbrand
Jan 5 at 19:37






$begingroup$
The "right" way to solve this is as Thomas Shelby wrote in his answer. However, you can almost intuitively see that the limit is $1$ even without using a calculator. Indeed, if $x$ is a huge positive number, say $10^{5863}$, the difference between $x^2$ and $x^2 + 1$ is extremely tiny. So the expression is essentially indistinguishable from ${xoversqrt{x^2}} = {xover x} = 1$.
$endgroup$
– Andreas Rejbrand
Jan 5 at 19:37












5 Answers
5






active

oldest

votes


















13












$begingroup$

Hint: Divide the numerator and denominator by $x $ and apply the limit.



$$frac{x}{sqrt{x^2 + 1}}=frac{1}{sqrt{1 + frac{1}{x^2}}}$$






share|cite|improve this answer











$endgroup$









  • 6




    $begingroup$
    In other words, actually do the division you see written down in $lim_{xtoinfty}frac{sqrt{x^2+1}}x$.
    $endgroup$
    – Arthur
    Jan 5 at 18:40





















11












$begingroup$

Hint



Simply use $${xover x+1}={xover sqrt{x^2+2x+1}}<{xover sqrt{x^2+1}}<1$$for large enough $x>0$.






share|cite|improve this answer









$endgroup$









  • 1




    $begingroup$
    Mostafa.Very nice+.
    $endgroup$
    – Peter Szilas
    Jan 5 at 19:01










  • $begingroup$
    But how so we know the limit is 1. Based on what you wrote couldnt the answer be 1/2
    $endgroup$
    – Milan Stojanovic
    Jan 5 at 19:05










  • $begingroup$
    @PeterSzilas thank you!
    $endgroup$
    – Mostafa Ayaz
    Jan 5 at 19:08






  • 3




    $begingroup$
    @MilanStojanovic No since $$lim_{xto infty}{xover x+1}=1$$and $$lim_{xto infty}1=1$$so the Squeeze theorem is applicable here....
    $endgroup$
    – Mostafa Ayaz
    Jan 5 at 19:09



















7












$begingroup$

By your own reasoning, you have the following:
$$limlimits_{xto infty} frac{x}{sqrt{x^2 + 1}}=limlimits_{xto infty} frac{sqrt{x^2 + 1}}{x}$$



Now, the left side is clearly the reciprocal of the right side, so we have:
$$limlimits_{xto infty} frac{x}{sqrt{x^2 + 1}}=frac{1}{limlimits_{xto infty} frac{x}{sqrt{x^2 + 1}}}$$



(Note that doing this manipulation assumes that $limlimits_{xto infty} frac{x}{sqrt{x^2 + 1}}$ converges to a real number. However, you can use the first derivative to show this is an always increasing function and then use basic algebra to show that $frac{x}{sqrt{x^2 + 1}} < 1$ for all $xinBbb{R}$. Thus, because this is a bounded, always increasing function, the limit as $xto infty$ must converge to some real number, so our assumption in this manipulation is valid.)



Cross-multiply:
$$left(limlimits_{xto infty} frac{x}{sqrt{x^2 + 1}}right)^2=1$$



Take the square root:
$$limlimits_{xto infty} frac{x}{sqrt{x^2 + 1}}=pm 1$$



However, it is easy to show that $frac{x}{sqrt{x^2 + 1}}>0$ for all $x > 0$. Therefore, there's no way the limit can be a negative number like $-1$. Thus, the only possibility we have left is $+1$, so:
$$limlimits_{xto infty} frac{x}{sqrt{x^2 + 1}}=1$$






share|cite|improve this answer









$endgroup$





















    4












    $begingroup$

    When computing the limit of rational functions, as is the case for $$lim_{x rightarrow infty} frac{x}{sqrt{x^2 +1}},$$ you want to divide the top and bottom by the highest degree in the denominator, which in this case is $x$. Since $x rightarrow +infty$, so $x$ is always positive (at least, near where we are worried about) I claim that $x = sqrt{x^2}$. So, if we divide the top and bottom by $x$, we get $$lim_{x rightarrow infty} frac{x}{sqrt{x^2 +1}} = lim_{x rightarrow infty} frac{1}{sqrt{1 + 1/x^2}}.$$ You should be able to compute the limit from here.



    Whenever you see a monomial in the numerator with the square root of a polynomial in the denominator, you should consider this method. Of course, keep in mind that you'll have to tweak it slightly if $x rightarrow -infty$! Try to see if you can figure out what would change in that case.






    share|cite|improve this answer











    $endgroup$





















      0












      $begingroup$

      Set $x = sinh t$. We have
      $$frac{x}{sqrt{x^2+1}}= frac{sinh t}{sqrt{1+sinh^2t}} = frac{sinh t}{cosh t} = tanh t$$



      $x to infty$ is equivalent to $ttoinfty$ so $$lim_{xtoinfty} frac{x}{sqrt{x^2+1}} = lim_{ttoinfty} tanh t = lim_{ttoinfty}frac{e^t - e^{-t}}{e^t+e^{-t}} = lim_{ttoinfty}frac{e^{2t}-1}{e^{2t}+1} = 1$$






      share|cite|improve this answer









      $endgroup$













        Your Answer





        StackExchange.ifUsing("editor", function () {
        return StackExchange.using("mathjaxEditing", function () {
        StackExchange.MarkdownEditor.creationCallbacks.add(function (editor, postfix) {
        StackExchange.mathjaxEditing.prepareWmdForMathJax(editor, postfix, [["$", "$"], ["\\(","\\)"]]);
        });
        });
        }, "mathjax-editing");

        StackExchange.ready(function() {
        var channelOptions = {
        tags: "".split(" "),
        id: "69"
        };
        initTagRenderer("".split(" "), "".split(" "), channelOptions);

        StackExchange.using("externalEditor", function() {
        // Have to fire editor after snippets, if snippets enabled
        if (StackExchange.settings.snippets.snippetsEnabled) {
        StackExchange.using("snippets", function() {
        createEditor();
        });
        }
        else {
        createEditor();
        }
        });

        function createEditor() {
        StackExchange.prepareEditor({
        heartbeatType: 'answer',
        autoActivateHeartbeat: false,
        convertImagesToLinks: true,
        noModals: true,
        showLowRepImageUploadWarning: true,
        reputationToPostImages: 10,
        bindNavPrevention: true,
        postfix: "",
        imageUploader: {
        brandingHtml: "Powered by u003ca class="icon-imgur-white" href="https://imgur.com/"u003eu003c/au003e",
        contentPolicyHtml: "User contributions licensed under u003ca href="https://creativecommons.org/licenses/by-sa/3.0/"u003ecc by-sa 3.0 with attribution requiredu003c/au003e u003ca href="https://stackoverflow.com/legal/content-policy"u003e(content policy)u003c/au003e",
        allowUrls: true
        },
        noCode: true, onDemand: true,
        discardSelector: ".discard-answer"
        ,immediatelyShowMarkdownHelp:true
        });


        }
        });














        draft saved

        draft discarded


















        StackExchange.ready(
        function () {
        StackExchange.openid.initPostLogin('.new-post-login', 'https%3a%2f%2fmath.stackexchange.com%2fquestions%2f3063053%2fis-there-a-way-to-find-this-limit-algebraically-lim-limits-x-to-infty-fra%23new-answer', 'question_page');
        }
        );

        Post as a guest















        Required, but never shown

























        5 Answers
        5






        active

        oldest

        votes








        5 Answers
        5






        active

        oldest

        votes









        active

        oldest

        votes






        active

        oldest

        votes









        13












        $begingroup$

        Hint: Divide the numerator and denominator by $x $ and apply the limit.



        $$frac{x}{sqrt{x^2 + 1}}=frac{1}{sqrt{1 + frac{1}{x^2}}}$$






        share|cite|improve this answer











        $endgroup$









        • 6




          $begingroup$
          In other words, actually do the division you see written down in $lim_{xtoinfty}frac{sqrt{x^2+1}}x$.
          $endgroup$
          – Arthur
          Jan 5 at 18:40


















        13












        $begingroup$

        Hint: Divide the numerator and denominator by $x $ and apply the limit.



        $$frac{x}{sqrt{x^2 + 1}}=frac{1}{sqrt{1 + frac{1}{x^2}}}$$






        share|cite|improve this answer











        $endgroup$









        • 6




          $begingroup$
          In other words, actually do the division you see written down in $lim_{xtoinfty}frac{sqrt{x^2+1}}x$.
          $endgroup$
          – Arthur
          Jan 5 at 18:40
















        13












        13








        13





        $begingroup$

        Hint: Divide the numerator and denominator by $x $ and apply the limit.



        $$frac{x}{sqrt{x^2 + 1}}=frac{1}{sqrt{1 + frac{1}{x^2}}}$$






        share|cite|improve this answer











        $endgroup$



        Hint: Divide the numerator and denominator by $x $ and apply the limit.



        $$frac{x}{sqrt{x^2 + 1}}=frac{1}{sqrt{1 + frac{1}{x^2}}}$$







        share|cite|improve this answer














        share|cite|improve this answer



        share|cite|improve this answer








        edited Jan 5 at 18:46

























        answered Jan 5 at 18:38









        Thomas ShelbyThomas Shelby

        2,453221




        2,453221








        • 6




          $begingroup$
          In other words, actually do the division you see written down in $lim_{xtoinfty}frac{sqrt{x^2+1}}x$.
          $endgroup$
          – Arthur
          Jan 5 at 18:40
















        • 6




          $begingroup$
          In other words, actually do the division you see written down in $lim_{xtoinfty}frac{sqrt{x^2+1}}x$.
          $endgroup$
          – Arthur
          Jan 5 at 18:40










        6




        6




        $begingroup$
        In other words, actually do the division you see written down in $lim_{xtoinfty}frac{sqrt{x^2+1}}x$.
        $endgroup$
        – Arthur
        Jan 5 at 18:40






        $begingroup$
        In other words, actually do the division you see written down in $lim_{xtoinfty}frac{sqrt{x^2+1}}x$.
        $endgroup$
        – Arthur
        Jan 5 at 18:40













        11












        $begingroup$

        Hint



        Simply use $${xover x+1}={xover sqrt{x^2+2x+1}}<{xover sqrt{x^2+1}}<1$$for large enough $x>0$.






        share|cite|improve this answer









        $endgroup$









        • 1




          $begingroup$
          Mostafa.Very nice+.
          $endgroup$
          – Peter Szilas
          Jan 5 at 19:01










        • $begingroup$
          But how so we know the limit is 1. Based on what you wrote couldnt the answer be 1/2
          $endgroup$
          – Milan Stojanovic
          Jan 5 at 19:05










        • $begingroup$
          @PeterSzilas thank you!
          $endgroup$
          – Mostafa Ayaz
          Jan 5 at 19:08






        • 3




          $begingroup$
          @MilanStojanovic No since $$lim_{xto infty}{xover x+1}=1$$and $$lim_{xto infty}1=1$$so the Squeeze theorem is applicable here....
          $endgroup$
          – Mostafa Ayaz
          Jan 5 at 19:09
















        11












        $begingroup$

        Hint



        Simply use $${xover x+1}={xover sqrt{x^2+2x+1}}<{xover sqrt{x^2+1}}<1$$for large enough $x>0$.






        share|cite|improve this answer









        $endgroup$









        • 1




          $begingroup$
          Mostafa.Very nice+.
          $endgroup$
          – Peter Szilas
          Jan 5 at 19:01










        • $begingroup$
          But how so we know the limit is 1. Based on what you wrote couldnt the answer be 1/2
          $endgroup$
          – Milan Stojanovic
          Jan 5 at 19:05










        • $begingroup$
          @PeterSzilas thank you!
          $endgroup$
          – Mostafa Ayaz
          Jan 5 at 19:08






        • 3




          $begingroup$
          @MilanStojanovic No since $$lim_{xto infty}{xover x+1}=1$$and $$lim_{xto infty}1=1$$so the Squeeze theorem is applicable here....
          $endgroup$
          – Mostafa Ayaz
          Jan 5 at 19:09














        11












        11








        11





        $begingroup$

        Hint



        Simply use $${xover x+1}={xover sqrt{x^2+2x+1}}<{xover sqrt{x^2+1}}<1$$for large enough $x>0$.






        share|cite|improve this answer









        $endgroup$



        Hint



        Simply use $${xover x+1}={xover sqrt{x^2+2x+1}}<{xover sqrt{x^2+1}}<1$$for large enough $x>0$.







        share|cite|improve this answer












        share|cite|improve this answer



        share|cite|improve this answer










        answered Jan 5 at 18:53









        Mostafa AyazMostafa Ayaz

        15.4k3939




        15.4k3939








        • 1




          $begingroup$
          Mostafa.Very nice+.
          $endgroup$
          – Peter Szilas
          Jan 5 at 19:01










        • $begingroup$
          But how so we know the limit is 1. Based on what you wrote couldnt the answer be 1/2
          $endgroup$
          – Milan Stojanovic
          Jan 5 at 19:05










        • $begingroup$
          @PeterSzilas thank you!
          $endgroup$
          – Mostafa Ayaz
          Jan 5 at 19:08






        • 3




          $begingroup$
          @MilanStojanovic No since $$lim_{xto infty}{xover x+1}=1$$and $$lim_{xto infty}1=1$$so the Squeeze theorem is applicable here....
          $endgroup$
          – Mostafa Ayaz
          Jan 5 at 19:09














        • 1




          $begingroup$
          Mostafa.Very nice+.
          $endgroup$
          – Peter Szilas
          Jan 5 at 19:01










        • $begingroup$
          But how so we know the limit is 1. Based on what you wrote couldnt the answer be 1/2
          $endgroup$
          – Milan Stojanovic
          Jan 5 at 19:05










        • $begingroup$
          @PeterSzilas thank you!
          $endgroup$
          – Mostafa Ayaz
          Jan 5 at 19:08






        • 3




          $begingroup$
          @MilanStojanovic No since $$lim_{xto infty}{xover x+1}=1$$and $$lim_{xto infty}1=1$$so the Squeeze theorem is applicable here....
          $endgroup$
          – Mostafa Ayaz
          Jan 5 at 19:09








        1




        1




        $begingroup$
        Mostafa.Very nice+.
        $endgroup$
        – Peter Szilas
        Jan 5 at 19:01




        $begingroup$
        Mostafa.Very nice+.
        $endgroup$
        – Peter Szilas
        Jan 5 at 19:01












        $begingroup$
        But how so we know the limit is 1. Based on what you wrote couldnt the answer be 1/2
        $endgroup$
        – Milan Stojanovic
        Jan 5 at 19:05




        $begingroup$
        But how so we know the limit is 1. Based on what you wrote couldnt the answer be 1/2
        $endgroup$
        – Milan Stojanovic
        Jan 5 at 19:05












        $begingroup$
        @PeterSzilas thank you!
        $endgroup$
        – Mostafa Ayaz
        Jan 5 at 19:08




        $begingroup$
        @PeterSzilas thank you!
        $endgroup$
        – Mostafa Ayaz
        Jan 5 at 19:08




        3




        3




        $begingroup$
        @MilanStojanovic No since $$lim_{xto infty}{xover x+1}=1$$and $$lim_{xto infty}1=1$$so the Squeeze theorem is applicable here....
        $endgroup$
        – Mostafa Ayaz
        Jan 5 at 19:09




        $begingroup$
        @MilanStojanovic No since $$lim_{xto infty}{xover x+1}=1$$and $$lim_{xto infty}1=1$$so the Squeeze theorem is applicable here....
        $endgroup$
        – Mostafa Ayaz
        Jan 5 at 19:09











        7












        $begingroup$

        By your own reasoning, you have the following:
        $$limlimits_{xto infty} frac{x}{sqrt{x^2 + 1}}=limlimits_{xto infty} frac{sqrt{x^2 + 1}}{x}$$



        Now, the left side is clearly the reciprocal of the right side, so we have:
        $$limlimits_{xto infty} frac{x}{sqrt{x^2 + 1}}=frac{1}{limlimits_{xto infty} frac{x}{sqrt{x^2 + 1}}}$$



        (Note that doing this manipulation assumes that $limlimits_{xto infty} frac{x}{sqrt{x^2 + 1}}$ converges to a real number. However, you can use the first derivative to show this is an always increasing function and then use basic algebra to show that $frac{x}{sqrt{x^2 + 1}} < 1$ for all $xinBbb{R}$. Thus, because this is a bounded, always increasing function, the limit as $xto infty$ must converge to some real number, so our assumption in this manipulation is valid.)



        Cross-multiply:
        $$left(limlimits_{xto infty} frac{x}{sqrt{x^2 + 1}}right)^2=1$$



        Take the square root:
        $$limlimits_{xto infty} frac{x}{sqrt{x^2 + 1}}=pm 1$$



        However, it is easy to show that $frac{x}{sqrt{x^2 + 1}}>0$ for all $x > 0$. Therefore, there's no way the limit can be a negative number like $-1$. Thus, the only possibility we have left is $+1$, so:
        $$limlimits_{xto infty} frac{x}{sqrt{x^2 + 1}}=1$$






        share|cite|improve this answer









        $endgroup$


















          7












          $begingroup$

          By your own reasoning, you have the following:
          $$limlimits_{xto infty} frac{x}{sqrt{x^2 + 1}}=limlimits_{xto infty} frac{sqrt{x^2 + 1}}{x}$$



          Now, the left side is clearly the reciprocal of the right side, so we have:
          $$limlimits_{xto infty} frac{x}{sqrt{x^2 + 1}}=frac{1}{limlimits_{xto infty} frac{x}{sqrt{x^2 + 1}}}$$



          (Note that doing this manipulation assumes that $limlimits_{xto infty} frac{x}{sqrt{x^2 + 1}}$ converges to a real number. However, you can use the first derivative to show this is an always increasing function and then use basic algebra to show that $frac{x}{sqrt{x^2 + 1}} < 1$ for all $xinBbb{R}$. Thus, because this is a bounded, always increasing function, the limit as $xto infty$ must converge to some real number, so our assumption in this manipulation is valid.)



          Cross-multiply:
          $$left(limlimits_{xto infty} frac{x}{sqrt{x^2 + 1}}right)^2=1$$



          Take the square root:
          $$limlimits_{xto infty} frac{x}{sqrt{x^2 + 1}}=pm 1$$



          However, it is easy to show that $frac{x}{sqrt{x^2 + 1}}>0$ for all $x > 0$. Therefore, there's no way the limit can be a negative number like $-1$. Thus, the only possibility we have left is $+1$, so:
          $$limlimits_{xto infty} frac{x}{sqrt{x^2 + 1}}=1$$






          share|cite|improve this answer









          $endgroup$
















            7












            7








            7





            $begingroup$

            By your own reasoning, you have the following:
            $$limlimits_{xto infty} frac{x}{sqrt{x^2 + 1}}=limlimits_{xto infty} frac{sqrt{x^2 + 1}}{x}$$



            Now, the left side is clearly the reciprocal of the right side, so we have:
            $$limlimits_{xto infty} frac{x}{sqrt{x^2 + 1}}=frac{1}{limlimits_{xto infty} frac{x}{sqrt{x^2 + 1}}}$$



            (Note that doing this manipulation assumes that $limlimits_{xto infty} frac{x}{sqrt{x^2 + 1}}$ converges to a real number. However, you can use the first derivative to show this is an always increasing function and then use basic algebra to show that $frac{x}{sqrt{x^2 + 1}} < 1$ for all $xinBbb{R}$. Thus, because this is a bounded, always increasing function, the limit as $xto infty$ must converge to some real number, so our assumption in this manipulation is valid.)



            Cross-multiply:
            $$left(limlimits_{xto infty} frac{x}{sqrt{x^2 + 1}}right)^2=1$$



            Take the square root:
            $$limlimits_{xto infty} frac{x}{sqrt{x^2 + 1}}=pm 1$$



            However, it is easy to show that $frac{x}{sqrt{x^2 + 1}}>0$ for all $x > 0$. Therefore, there's no way the limit can be a negative number like $-1$. Thus, the only possibility we have left is $+1$, so:
            $$limlimits_{xto infty} frac{x}{sqrt{x^2 + 1}}=1$$






            share|cite|improve this answer









            $endgroup$



            By your own reasoning, you have the following:
            $$limlimits_{xto infty} frac{x}{sqrt{x^2 + 1}}=limlimits_{xto infty} frac{sqrt{x^2 + 1}}{x}$$



            Now, the left side is clearly the reciprocal of the right side, so we have:
            $$limlimits_{xto infty} frac{x}{sqrt{x^2 + 1}}=frac{1}{limlimits_{xto infty} frac{x}{sqrt{x^2 + 1}}}$$



            (Note that doing this manipulation assumes that $limlimits_{xto infty} frac{x}{sqrt{x^2 + 1}}$ converges to a real number. However, you can use the first derivative to show this is an always increasing function and then use basic algebra to show that $frac{x}{sqrt{x^2 + 1}} < 1$ for all $xinBbb{R}$. Thus, because this is a bounded, always increasing function, the limit as $xto infty$ must converge to some real number, so our assumption in this manipulation is valid.)



            Cross-multiply:
            $$left(limlimits_{xto infty} frac{x}{sqrt{x^2 + 1}}right)^2=1$$



            Take the square root:
            $$limlimits_{xto infty} frac{x}{sqrt{x^2 + 1}}=pm 1$$



            However, it is easy to show that $frac{x}{sqrt{x^2 + 1}}>0$ for all $x > 0$. Therefore, there's no way the limit can be a negative number like $-1$. Thus, the only possibility we have left is $+1$, so:
            $$limlimits_{xto infty} frac{x}{sqrt{x^2 + 1}}=1$$







            share|cite|improve this answer












            share|cite|improve this answer



            share|cite|improve this answer










            answered Jan 5 at 18:46









            Noble MushtakNoble Mushtak

            15.2k1735




            15.2k1735























                4












                $begingroup$

                When computing the limit of rational functions, as is the case for $$lim_{x rightarrow infty} frac{x}{sqrt{x^2 +1}},$$ you want to divide the top and bottom by the highest degree in the denominator, which in this case is $x$. Since $x rightarrow +infty$, so $x$ is always positive (at least, near where we are worried about) I claim that $x = sqrt{x^2}$. So, if we divide the top and bottom by $x$, we get $$lim_{x rightarrow infty} frac{x}{sqrt{x^2 +1}} = lim_{x rightarrow infty} frac{1}{sqrt{1 + 1/x^2}}.$$ You should be able to compute the limit from here.



                Whenever you see a monomial in the numerator with the square root of a polynomial in the denominator, you should consider this method. Of course, keep in mind that you'll have to tweak it slightly if $x rightarrow -infty$! Try to see if you can figure out what would change in that case.






                share|cite|improve this answer











                $endgroup$


















                  4












                  $begingroup$

                  When computing the limit of rational functions, as is the case for $$lim_{x rightarrow infty} frac{x}{sqrt{x^2 +1}},$$ you want to divide the top and bottom by the highest degree in the denominator, which in this case is $x$. Since $x rightarrow +infty$, so $x$ is always positive (at least, near where we are worried about) I claim that $x = sqrt{x^2}$. So, if we divide the top and bottom by $x$, we get $$lim_{x rightarrow infty} frac{x}{sqrt{x^2 +1}} = lim_{x rightarrow infty} frac{1}{sqrt{1 + 1/x^2}}.$$ You should be able to compute the limit from here.



                  Whenever you see a monomial in the numerator with the square root of a polynomial in the denominator, you should consider this method. Of course, keep in mind that you'll have to tweak it slightly if $x rightarrow -infty$! Try to see if you can figure out what would change in that case.






                  share|cite|improve this answer











                  $endgroup$
















                    4












                    4








                    4





                    $begingroup$

                    When computing the limit of rational functions, as is the case for $$lim_{x rightarrow infty} frac{x}{sqrt{x^2 +1}},$$ you want to divide the top and bottom by the highest degree in the denominator, which in this case is $x$. Since $x rightarrow +infty$, so $x$ is always positive (at least, near where we are worried about) I claim that $x = sqrt{x^2}$. So, if we divide the top and bottom by $x$, we get $$lim_{x rightarrow infty} frac{x}{sqrt{x^2 +1}} = lim_{x rightarrow infty} frac{1}{sqrt{1 + 1/x^2}}.$$ You should be able to compute the limit from here.



                    Whenever you see a monomial in the numerator with the square root of a polynomial in the denominator, you should consider this method. Of course, keep in mind that you'll have to tweak it slightly if $x rightarrow -infty$! Try to see if you can figure out what would change in that case.






                    share|cite|improve this answer











                    $endgroup$



                    When computing the limit of rational functions, as is the case for $$lim_{x rightarrow infty} frac{x}{sqrt{x^2 +1}},$$ you want to divide the top and bottom by the highest degree in the denominator, which in this case is $x$. Since $x rightarrow +infty$, so $x$ is always positive (at least, near where we are worried about) I claim that $x = sqrt{x^2}$. So, if we divide the top and bottom by $x$, we get $$lim_{x rightarrow infty} frac{x}{sqrt{x^2 +1}} = lim_{x rightarrow infty} frac{1}{sqrt{1 + 1/x^2}}.$$ You should be able to compute the limit from here.



                    Whenever you see a monomial in the numerator with the square root of a polynomial in the denominator, you should consider this method. Of course, keep in mind that you'll have to tweak it slightly if $x rightarrow -infty$! Try to see if you can figure out what would change in that case.







                    share|cite|improve this answer














                    share|cite|improve this answer



                    share|cite|improve this answer








                    edited Jan 5 at 18:52









                    Noble Mushtak

                    15.2k1735




                    15.2k1735










                    answered Jan 5 at 18:51









                    kkckkc

                    1108




                    1108























                        0












                        $begingroup$

                        Set $x = sinh t$. We have
                        $$frac{x}{sqrt{x^2+1}}= frac{sinh t}{sqrt{1+sinh^2t}} = frac{sinh t}{cosh t} = tanh t$$



                        $x to infty$ is equivalent to $ttoinfty$ so $$lim_{xtoinfty} frac{x}{sqrt{x^2+1}} = lim_{ttoinfty} tanh t = lim_{ttoinfty}frac{e^t - e^{-t}}{e^t+e^{-t}} = lim_{ttoinfty}frac{e^{2t}-1}{e^{2t}+1} = 1$$






                        share|cite|improve this answer









                        $endgroup$


















                          0












                          $begingroup$

                          Set $x = sinh t$. We have
                          $$frac{x}{sqrt{x^2+1}}= frac{sinh t}{sqrt{1+sinh^2t}} = frac{sinh t}{cosh t} = tanh t$$



                          $x to infty$ is equivalent to $ttoinfty$ so $$lim_{xtoinfty} frac{x}{sqrt{x^2+1}} = lim_{ttoinfty} tanh t = lim_{ttoinfty}frac{e^t - e^{-t}}{e^t+e^{-t}} = lim_{ttoinfty}frac{e^{2t}-1}{e^{2t}+1} = 1$$






                          share|cite|improve this answer









                          $endgroup$
















                            0












                            0








                            0





                            $begingroup$

                            Set $x = sinh t$. We have
                            $$frac{x}{sqrt{x^2+1}}= frac{sinh t}{sqrt{1+sinh^2t}} = frac{sinh t}{cosh t} = tanh t$$



                            $x to infty$ is equivalent to $ttoinfty$ so $$lim_{xtoinfty} frac{x}{sqrt{x^2+1}} = lim_{ttoinfty} tanh t = lim_{ttoinfty}frac{e^t - e^{-t}}{e^t+e^{-t}} = lim_{ttoinfty}frac{e^{2t}-1}{e^{2t}+1} = 1$$






                            share|cite|improve this answer









                            $endgroup$



                            Set $x = sinh t$. We have
                            $$frac{x}{sqrt{x^2+1}}= frac{sinh t}{sqrt{1+sinh^2t}} = frac{sinh t}{cosh t} = tanh t$$



                            $x to infty$ is equivalent to $ttoinfty$ so $$lim_{xtoinfty} frac{x}{sqrt{x^2+1}} = lim_{ttoinfty} tanh t = lim_{ttoinfty}frac{e^t - e^{-t}}{e^t+e^{-t}} = lim_{ttoinfty}frac{e^{2t}-1}{e^{2t}+1} = 1$$







                            share|cite|improve this answer












                            share|cite|improve this answer



                            share|cite|improve this answer










                            answered Jan 5 at 21:08









                            mechanodroidmechanodroid

                            27.1k62446




                            27.1k62446






























                                draft saved

                                draft discarded




















































                                Thanks for contributing an answer to Mathematics Stack Exchange!


                                • Please be sure to answer the question. Provide details and share your research!

                                But avoid



                                • Asking for help, clarification, or responding to other answers.

                                • Making statements based on opinion; back them up with references or personal experience.


                                Use MathJax to format equations. MathJax reference.


                                To learn more, see our tips on writing great answers.




                                draft saved


                                draft discarded














                                StackExchange.ready(
                                function () {
                                StackExchange.openid.initPostLogin('.new-post-login', 'https%3a%2f%2fmath.stackexchange.com%2fquestions%2f3063053%2fis-there-a-way-to-find-this-limit-algebraically-lim-limits-x-to-infty-fra%23new-answer', 'question_page');
                                }
                                );

                                Post as a guest















                                Required, but never shown





















































                                Required, but never shown














                                Required, but never shown












                                Required, but never shown







                                Required, but never shown

































                                Required, but never shown














                                Required, but never shown












                                Required, but never shown







                                Required, but never shown







                                Popular posts from this blog

                                How do I know what Microsoft account the skydrive app is syncing to?

                                When does type information flow backwards in C++?

                                Grease: Live!